LSAT and Law School Admissions Forum

Get expert LSAT preparation and law school admissions advice from PowerScore Test Preparation.

User avatar
 Dave Killoran
PowerScore Staff
  • PowerScore Staff
  • Posts: 5850
  • Joined: Mar 25, 2011
|
#88070
Complete Question Explanation
(The complete setup for this game can be found here: lsat/viewtopic.php?f=166&t=8547)

The correct answer choice is (E).

Although this appears to be a question where you should immediately consult the sequence, do not forget that prior work is perfect for solving a question like this one.

From the hypothetical produced in question #13, we know that G can be third. Thus, we can eliminate answer choices (A) and (D).

From the question stem in question #14, we know that F can be third. Thus, we can eliminate answer choice (C).

From the correct answer choice in question #16, we know that H can be third. This also eliminates answer choice (A).

From the hypotheticals produced in question #17, we know that R (and G) can be third. This eliminates answer choice (B).

Only answer choice (E) remains, and thus answer choice (E) is correct.

Get the most out of your LSAT Prep Plus subscription.

Analyze and track your performance with our Testing and Analytics Package.